Neurology PreTest Review
A 3-month-old child has a rapid regression of psychomotor func- tion and loss of sight. There is increased urinary excretion of N-acetyl-L- aspartic acid. A preliminary diagnosis of Canavan's disease (Canavan-van Bogaert-Bertrand disease; spongy degeneration of infancy) is made. This is a demyelinating disease that produces retardation in infants, is inherited in an autosomal recessive pattern, and results in which of the following? a. Anencephaly b. Microcephaly c. Porencephaly d. Macrocephaly e. Dolichocephaly
278. The answer is d. Canavan's disease may pro- duce developmental regression at about 6 months of age. The infant devel- ops extensor posturing and rigidity. Myoclonic seizures may develop. Underlying the disease is a defect in N-acetylaspartic acid metabolism. Ele- vated levels of this material can be detected in the blood and urine, but ele- vated levels in the brain establish the diagnosis. Changes in brain white matter are widespread and may result in a spongiform appearance. There is an increase in brain volume and weight
A 4-year-old Jewish child has a history of poor sucking at birth, as well as multiple respiratory infections during childhood. He is of short stature and has not been able to eat due to progressive vomiting. On examination, strength is normal, but he is hyporeflexic. There is sensory dissociation, with loss of pain and temperature sensation and preservation of tactile and vibratory sense. The corneas are ulcerated, pupils do not react, and he has orthostatic hypotension. Dx?
Familial dysautonomia, or Riley- Day disease, is an autosomal recessive disorder that affects primarily Jewish children. It is characterized by a small-fiber neuropathy affecting both myelinated and unmyelinated small fibers, thereby *causing an impairment of pain and temperature sensation*. Sympathetic and parasympathetic ganglia are also affected, *causing the autonomic features*. The autonomic manifestations may include loss of tears on crying, corneal ulceration, absence of pupillary reactivity, poor temperature regulation, excessive perspiration, abnormalities of blood pressure control, dysphagia, recurrent vomiting, and gastric and intestinal dilation. There is absence of the papillae of the tongue. There is no effective treatment other than symptomatic.
A 38-year-old miner develops a shuffling gait, tremor, and drooling. His speech is difficult to understand and trails off in volume until it is inaudible. He consults a physician because of easy fatigability and frequent falls. Cogwheel rigidity is evident in his arms and legs. His tremor is most evident when his limbs are at rest. Dx?
Manganese inhalation by miners produces a clinical picture similar to that seen with hepatolenticular degeneration (Wilson's disease). Parkinsonism is the most prominent feature, but axial rigidity and dystonia may also develop. Neuronal loss is evident in several areas of the brain, including the globus pallidus, puta- men, caudate, hypothalamus, and cerebellum. Treatment with L-dopa is usually less effective with this heavy metal injury than it is with Parkinson's disease. Agents more likely to produce parkinsonism in the general population include phenothiazines, butyrophenones, and metoclopramide. Metoclopramide (Reglan) is used increasingly after gastrointestinal surgery to manage nausea and other signs of gastrointestinal irritability. Although most physicians do ask about exposure to reserpine-like medications or phenothiazines, other drugs that may cause parkinsonism in susceptible persons are sometimes overlooked.
A 34-year-old investment banker has intermittent episodes of vertigo associated with a feeling of fullness in his right ear. These last for several hours. He has had progressive hearing loss in the right ear. There are no other symptoms. He takes no medications and has no history of head trauma.
Ménière's disease is characterized by repeated brief episodes of fullness in the ear, tinnitus, hearing loss, and severe vertigo. The episodes may last from hours to days. Attacks may be so severe that they cause the patient to fall to the ground due to severe dis- equilibrium. The cause is generally idiopathic, but is thought to relate to distension of the semicircular canal and an increase in the volume of the endolymphatic fluid. For this reason, the condition has been called endo- lymphatic hydrops. Treatment is generally with salt restriction and diuretics. Surgery with endolymphatic shunts is of unproven value.
A 29-year-old man with a history of febrile seizures as a child has developed medication refractory complex partial seizures within the past 2 years. An MRI reveals the abnormality indicated by the arrow. Which of the following is true regarding this condition? a. This patient may benefit from a neurosurgical procedure b. The patient will probably die within 2 years c. The seizures will most likely stop with further medication titration d. A head CT should be performed e. A cerebral angiogram may confirm the diagnosis
The answer is a. (Bradley pp 1972-1973.) The history and MRI are typical for *mesial temporal sclerosis (MTS)*. The arrow in the MRI is specifically pointing at the sclerotic right hippocampus. *This is the most common cause of intractable complex partial seizures in adults*. The prognosis for improved seizure control with additional medications is poor; however surgical resection of the right anterior temporal lobe may produce seizure freedom in up to 80% of cases. If this patient had a high-grade malignant brain tumor, he would probably die within 2 years. A cerebral angiogram may confirm the diagnosis of a vascular malformation.
A patient with intractable complex partial seizures due to cortical dys- plasia undergoes left temporal lobectomy. He is most likely to develop which of the following problems after surgery? a. Right superior quadrantanopsia b. Right inferior quadrantanopsia c. Right homonymous hemianopsia d. Right hand weakness e. Aphasia
The answer is a. (Patten, p 25.) The most common complication of temporal lobectomy is a visual field defect due to interruption of fibers from the optic tracts passing over the temporal horn of the lateral ventricles. Superior quadrantanopsia is more common than hemianopsia. Some deficits may improve if the injury does not completely damage the nerves. Language deficits, particularly dysnomia, occur less frequently. Hemiparesis is uncommon (<2%), because the surgery is performed at a distance from the motor fibers of the corticospinal tract. Other neurological problems that can occur include diplopia due to extraocular nerve deficits, and facial paresis.
The peripheral neuropathy that would be expected to be seen in a patient with Friedreich's disease develops in part because of degeneration in which of the following? a. Dorsal root ganglia b. Spinocerebellar tracts c. Anterior horn cells d. Clarke's column e. Posterior columns
The answer is a. Degenerative changes in the peripheral nerves of patients with Friedreich's disease have several bases. Loss of cells in the dorsal root ganglia makes a major contribution to this phenomenon. Additional sites of degeneration that affect the sensory system include the substantia gelatinosa (Lissauer's tract) of the posterior horn and the dorsal roots themselves. The peripheral neuropathy is responsible for the hyporeflexia that is invariably found in the legs of affected persons.
A 4-year-old boy presents with ataxia, lethargy, and obstructive hydrocephalus.
The answer is a. (Swaiman, pp 1065-1067.) Medulloblastomas are one of the most common CNS tumors of childhood. They typically develop in the cerebellum, causing ataxia. Astrocytomas may also occur in children infratentorially, primarily in the cerebellum and brainstem. In either loca- tion, hydrocephalus may develop because of obstruction at the level of the fourth ventricle. The astrocytoma that develops in the cerebellum is usu- ally cystic. Medulloblastomas are invariably infratentorial, at least initially. They may extend supratentorially or become disseminated supratentorially through seeding of cells carried in the CSF. Ependymomas, another com- mon tumor type in children, are derived from the lining of the ventricles and also carry the risk of hydrocephalus and seeding throughout the CNS.
A 65-year-old man was diagnosed with lung cancer 6 months ago. Over the past 2 months, he has had worsening severe proximal muscle weakness. He is most likely to have which of the following? a. Dermatomyositis b. Trichinosis c. Multiple sclerosis (MS) d. Progressive multifocal leukoencephalopathy (PML) e. Myasthenia gravis
The answer is a. (Victor, p 1484.) Dermatomyositis occurs as a paraneoplastic syndrome in about 15% of cases overall. Among those over age 40, the proportion of paraneoplastic cases increases to 40% for women and 66% for men. Tumors underlying dermatomyositis may develop in the lungs, ovaries, gastrointestinal tract, breasts, or other organs, but the CNS is generally not the site of a tumor associated with dermatomyositis. Because of the higher probability of malignancy in adults with dermatomyositis, patients diagnosed with this inflammatory disease should routinely undergo a variety of diagnostic studies, including rectal and breast examinations, periodic screens for occult blood in the stool, and hemograms. Sputum cytologies and chest x-rays, as well as urine cytologic studies, are recom- mended by some physicians. Both PML and MS are strictly CNS diseases. Trichinosis is a parasitic disease that involves skeletal muscle and may produce substantial weakness, but it is not associated with any tumors.
A 51-year-old factory worker has noticed progressive weakness over the past year. Examination and testing reveal a painless largely motor peripheral neuropathy. Which of the following agents is most likely to be etiologic in this case? a. Lead b. Manganese c. Thallium d. Cyanide e. Mercury
The answer is a. (Victor, pp 1393-1394.) Lead poisoning, especially in adults, produces a painless neuropathy often targeting the radial nerve and resulting in a wrist-drop. Lead poisoning in children is likely to produce increased intracranial pressure and cognitive dysfunction. Thallium poisoning may produce hair loss, stupor, gastrointestinal distress, seizures, and headaches, as well as a painful, symmetric, primarily sensory neuropathy. Manganese is also a toxin, but long-term exposure to this metal may produce parkinsonism rather than a sensory neuropathy. Cyanide was long regarded as the cause of an optic neuropathy, but this lethal toxin has probably been unjustly ascribed this capability. Mercury poisoning may produce a sensory neuropathy, but it is generally associated with paresthesias rather than dysesthesias.
A 22-year-old male abuser of intravenous heroin has been having severe headaches during sexual intercourse. Within a few minutes of one headache, he develops right-sided weakness and becomes stuporous. His neurologic examination reveals neck stiffness as well as right arm and face weakness. An unenhanced emergency CT scan reveals a lesion of 3 to 4 cm in the cortex of the left parietal lobe. The addition of contrast enhancement reveals two other smaller lesions in the right frontal lobe but does not alter the appearance of the lesion in the left parietal lobe. Which of the follow- ing diagnostic studies is most likely to establish the basis for this patient's neurologic deficits? a. HIV antibody testing b. Cerebrospinal fluid (CSF) examination c. Electroencephalography d. Nerve conduction studies e. Cardiac catheterization
The answer is b. This young man almost certainly has numerous problems associated with his intravenous drug abuse, but the cause of his current problems is most likely bleeding from a *mycotic aneurysm*. Aneurysms are *especially likely to bleed during exertion*, such as that associated with sexual intercourse or defecation. The fact that the lesion appeared largely the same on unenhanced and enhanced CT scans suggests that it is a hematoma. HIV antibody testing might reveal evidence of exposure to HIV, but, aside from establishing that the patient was at increased risk of opportunistic infections, that test would provide little insight into the cause of the acute neurologic syndrome. The CSF would be expected to be xanthochromic (yellow), with many (>20/μL) red blood cells (RBCs), or grossly bloody, thereby providing evidence of a recent subarachnoid hemorrhage. Electroencephalography would undoubtedly reveal an asymmetric pattern associated with the left hemispheric lesion, but this too would provide little insight into the cause of the problem. Nerve conduction studies would not clarify the basis for a lesion of the central nervous system, because they examine only structures of the peripheral nervous system. Cardiac catheterization might reveal valvular abnormalities, but these need not be associated with disease of the central nervous system.
A 67-year-old woman has a 2-month history of progressive gait dis- turbance. On exam, she has dysmetria of the limbs; a wide-based, unsteady gait; and hypermetric saccades. A hard, firm breast lump is discovered.
The answer is a. (Victor, pp 724-725.) Paraneoplastic cerebellar degeneration (PCD) is characterized by subacute and relentlessly progres- sive ataxia, dysarthria, and nystagmus. Myoclonus, opsoclonus (irregular jerking of the eyes in all directions), diplopia, vertigo, and hearing loss may also occur. Imaging may eventually reveal cerebellar atrophy, and pathol- ogy will disclose loss of Purkinje cells in the cerebellum as the primary abnormality. The most common associated tumor types are small cell car- cinoma of the lung, ovarian carcinoma, and lymphoma, in that order. Approximately 50% of patients may harbor anti-Purkinje cell antibodies (called anti-Yo antibodies), and these are especially commonly found in women with breast cancer or other gynecologic malignancies. Interestingly, the symptoms of PCD often precede the symptoms of the underlying tumor itself, leading to speculation that the immune reaction that damages the nervous system may, in fact, be protective against the tumor.
A 53-year-old woman is unable to stop blinking forcefully, and has frequent grimacing movements of the face. At times she protrudes her tongue against her will. She has never taken any medications.
The answer is a. *Meigs' syndrome* is a form of focal dystonia characterized by blepharospasm, forceful jaw opening, lip retraction, neck contractions, and tongue thrusting. Sometimes these features are produced by phenothiazine or butyrophenone use, but they may also occur idiopathically, more often in women than men, with onset in the sixth decade. Botulinum toxin injection has been more effective in treatment than any oral medication.
A 7-year-old girl acutely develops horizontal diplopia that worsens over the course of a few days. Examination reveals that the double vision is exacerbated by leftward gaze. Red glass testing reveals that the "false" image is from the left eye. She is most likely to have which of the following? a. Pontine glioma b. Medullary glioma c. Mesencephalic infarction d. Pontine infarction e. Medullary infarction
The answer is a. An abducens dysfunction with lateral rectus palsy may develop in children with increased intracranial pressure or with direct damage to the brainstem. With a brainstem glioma, both brainstem damage and increased intracranial pressure may develop secondary to the tumor. The adult who develops an acute abducens palsy is also at high risk for tumor. Metastatic lesions from the nasopharynx are especially likely in the adult, but vascular disease is also a significant cause of ocular motor dysfunction in adults, especially in the elderly.
Most of the organisms found in brain abscesses are which of the following? a. Streptococcal b. Staphylococcal c. Bacteroides spp. d. Proteus spp. e. Pseudomonas spp.
The answer is a. Both aerobic and anaerobic streptococcal bacteria occur in more than half of all brain abscesses. Staphylococcus aureus most often occurs in patients who have had penetrating head wounds or have undergone neurosurgical procedures. Enteric bacteria (e.g., E. coli, Proteus, and Pseudomonas) account for twice as many abscesses as S. aureus.
In Tay-Sachs disease, the enzymatic abnormality responsible for the neurologic deficits is deficiency of which of the following? a. Hexosaminidase A b. Glucocerebrosidase c. Phosphofructokinase d. Glucose phosphorylase e. Sphingomyelinase
The answer is a. Children with Tay-Sachs disease die prematurely and exhibit mental retardation, seizures, and blindness. This is a ganglioside storage disease that occurs more commonly in Ashkenazi Jews than in the general population. The early-onset form will produce macrocephaly and a cherry red spot in the fundus.
A 60-year-old right-handed man underwent heart transplantation 2 weeks ago for severe ischemic cardiomyopathy. He had an uneventful post- operative course and went home after 1 week. He is now readmitted from an outside hospital where he was admitted with headaches, increasing confusion, and a generalized seizure. He relates that he has had difficulty seeing for several days. On exam, he has a blood pressure of 180/100. His pupils are equal and reactive, but he has difficulty reading and finding objects presented to him. Motor and sensory function are normal. An MRI shows several areas of T2 signal abnormality in the occipital and parietal lobe white matter bilaterally. A diffusion-weighted MRI sequence, sensitive to the changes of acute infarction, is negative. This patient's history, exam, and laboratory findings are most consistent with which of the following diagnoses? a. Cyclosporine toxicity b. Steroid psychosis c. Occipital lobe infarction d. Ischemic optic neuropathies e. Retinal detachment
The answer is a. Cyclosporine and tacrolimus (FK 506) may both induce a syndrome resembling hypertensive encephalopathy, which has been called by some reversible posterior leukoencephalopathy, although it involves more than white matter and may also occur in the anterior frontal regions. In the setting of cyclosporine use, patients may develop headache, visual dysfunction related to occipital lobe dysfunction, confusion, and seizures. Usually there is associated hypertension. The visual loss may include cortical blindness or scotomas. Imaging may show bilateral, more or less symmetrical signal changes in the white matter and occasionally the cortex of the occipital and parietal lobes.
Magnetic resonance imaging scan of a 19-year-old woman following a motor vehicle accident shows multiple foci of punctate hemorrhage. These are most likely indicative of which of the following? a. Diffuse axonal injury (DAI) b. Uncontrolled hypertension c. Amyloid angiopathy d. Ischemic infarction e. Coagulopathy
The answer is a. Diffuse axonal injury is the most common cause of coma in the head-injured patient without an intracranial mass lesion. It is characterized pathologically by diffusely spread axonal swellings affecting the white matter, corpus callosum, and upper brainstem. These foci are usually hemorrhagic. The etiology is thought to be due to *shearing forces on axons in certain susceptible regions of the brain*, notably those that are particularly vulnerable to rotational forces, such as the subcortical white matter, corpus callosum, and upper brainstem. Uncontrolled hypertension may occur in patients with HTN, but would be unlikely to produce this pattern of injury. Amyloid angiopathy causes multiple hemorrhages, but affects elderly patients. The decreased cerebral perfusion pressure associated with brain swelling and increased intracranial pressure could cause ischemic infarction, but this would not be expected to give this appearance on MRI. Coagulopathies also occur in up to 20% of patients.
A 34-year-old man develops progressive depression and memory impairment over the course of 6 months. His initial neurologic evaluation reveals a metabolic acidosis associated with his dementia. His liver is firm and his spleen appears to be slightly enlarged. He has tremor and rigidity in his arms and walks with relatively little swing in his arms. His blink is substantially reduced, which gives him the appearance of staring. An MRI of the brain reveals some atrophy of the putamen and globus pallidus. His CSF is normal. His EEG is unremarkable. Dx?
The answer is a. Hepatolenticular degeneration (Wilson's disease) often becomes symptomatic in the second or third decade of life, but its initial presentation may be delayed until the fourth or fifth decade. *Renal tubular acidosis develops along with hepatic fibrosis*. Systemic problems include heart and lung damage, but most patients become most symptomatic from their brain and liver disease. Dementia is progressive if the patient is not treated. Hepatic disease will progress to hepatic failure if the patient is left untreated. Appropriate treatment includes the chelating agent penicillamine, which depletes the body of copper.
A 35-year-old man with multiple sclerosis presents with blurry vision. Examination reveals that the medial rectus muscle fails to move synchronously with the contralateral lateral rectus muscle on attempted gaze to either size. When each eye is tested individually, medial rectus function is relatively preserved. In addition, prominent nystagmus is present in the abducting eye. Evidence of internuclear ophthalmoplegia (INO) indicates which of the following? a. A mesencephalic or pontine injury b. Thalamic hemorrhage c. Cerebellar dysfunction d. Cortical injury in the frontal eye fields e. Medullary infarction
The answer is a. In the MLF syndrome, the patient has incomplete adduction ipsilateral to the lesion in the MLF on conjugate lateral gaze. On attempted conjugate lateral gaze away from the side of the lesion, the patient has nystagmus in the abducting eye. The fast component of the nystagmus is directed temporally.
A 37-year-old man has difficulty relaxing his grip on his golf club after putting. He also is excessively somnolent. Examination reveals early cataract development, testicular atrophy, and baldness. His family says that he has become increasingly stubborn and hostile over the past 3 years. His electrocardiogram (ECG) reveals a minor conduction defect. An electro- myogram (EMG) will probably reveal which of the following? a. Repetitive discharges with minor stimulation b. Polyphasic giant action potentials c. Fasciculations d. Fibrillations e. Positive waves
The answer is a. Men with myotonic dystrophy characteristically exhibit problems with relaxing their grip, hypersomnolence, premature baldness, testicular atrophy, and cataracts. The EMG pattern displayed by these patients is often referred to as the dive bomber pattern because of the characteristic sound produced when the evoked action potentials are heard. The cardiac defect that evolves in these persons usually requires pacemaker implantation to avoid sudden death. Psychiatric problems also develop in many patients with myotonic dystrophy, but their basis is unknown.
A 23-year-old woman has had 1 week of worsening facial pain. She describes it as an intense shooting pain that comes and goes. It is present only on her right face. Which of the following is most likely to be this patient's underlying problem? a. Multiple sclerosis b. Tolosa-Hunt syndrome c. Migraine d. Anterior communicating artery aneurysm e. Falx meningioma
The answer is a. Multiple sclerosis is often associated with trigeminal neuralgia, which is then termed symptomatic trigeminal neuralgia because it occurs as a symptom of another illness. Other causes of symptomatic trigeminal neuralgia include basilar artery aneurysms, acoustic schwannomas, and posterior fossa meningiomas, all of which may cause injury to the fifth cranial nerve by compression. The Tolosa-Hunt syndrome is a presumably inflammatory disorder that produces ophthalmoplegia associated with headache and loss of sensation over the forehead. Pupillary function is usually spared, and the site of pathology is believed to be in the superior orbital fissure or the cavernous sinus. It is usually not associated with trigeminal neuralgia.
A 23-year-old woman awakens with bilateral leg weakness and numbness, urinary retention, and impaired bowel control. She has had several episodes of blurred vision over the previous 2 years, but these had always been attributed to idiopathic papillitis. Dx?
The answer is a. Neuromyelitis optica produces signs and symptoms of *bilateral optic neuritis in association with a transverse myelitis*. The paraparesis, bladder and bowel dysfunction, and sensory deficit signal a transverse myelitis—that is, an inflammatory demyelinating lesion that transects much of the spinal cord. In some cases, the pathology shows a necrotizing process in the spinal cord. All of these problems may develop with MS, but cerebellar involvement, more scat- tered cerebral involvement, and a generally less circumscribed pattern of deficits are more likely. Adults are especially likely to develop a pattern more typical of relapsing-remitting MS after an initial episode of neuromyelitis optica. Children presenting with neuromyelitis optica may have no other signs or symptoms of demyelination.
Which of the following types of visual field cuts is most often seen with vitamin B12 deficiency? a. Centrocecal scotoma b. Homonymous hemianopsia c. Bitemporal hemianopsia d. Binasal hemianopsia e. Hemianopsia with central sparing
The answer is a. The blind spot that normally occurs in each eye enlarges and extends temporally to involve central vision in patients with chronic vitamin B12 deficiency. This is similar to the blind spot that is associated with alcohol and tobacco excess, a problem called tobacco-alcohol amblyopia. Tobacco-alcohol amblyopia also seems to develop because of a vitamin B deficiency, but the deficiency is presumed to be of thiamine rather than of cobalamin.
A 26-year-old man diagnosed with von Hippel-Lindau syndrome has a postcontrast computed tomography (CT) scan that reveals a cyst and two smaller masses in the left cerebellar hemisphere. Which of the following is the best recommendation to this patient? a. Submit to surgical resection of the cerebellar lesions as soon as possible b. Submit to radiation therapy of the cerebellar lesions immediately c. Have follow-up MRI to look for involution of the lesions d. Have a diagnostic lumbar puncture to look for evidence of parasitic infestation of the brain e. Have a needle biopsy of the cerebellum to establish the histology of the cystic lesion
The answer is a. The cystic lesion and the other cerebellar lesions are most likely *hemangioblastomas*. These hemangioblastomas *often bleed and produce potentially lethal intracranial hematomas*. Radiation therapy and needle biopsies would increase the risk of bleeding. Rather than spontaneously involuting, these lesions generally enlarge and become more unstable as time passes. Intracerebellar hemorrhage is increasingly likely as time passes.
With β-glucosidase deficiency, the affected child is likely to exhibit abnormal accumulations of which of the following? a. Glucosylceramide b. GM2 ganglioside c. Galactosyl sulfatides d. Sphingomyelin e. Trihexosylceramide
The answer is a. The disease responsible for the accumulation of *glucosylceramide is Gaucher's disease*. Gaucher's disease is inherited as an autosomal recessive trait and may be diagnosed by demonstrating deficient glucocerebrosidase in fibroblasts or leukocytes. The severity of disease varies from non-neuronopathic types to acute infantile neuronopathic disease. Gaucher's disease produces *hepatosplenomegaly and may cause lethal CNS disease*. It is one of a collection of storage diseases called sphingolipidoses, which include Niemann-Pick disease, Krabbe's disease, and Fabry's disease. Fabry's disease involves the accumulation of another ceramide, trihexosylceramide. All the sphingolipids are nothing more than lipids that contain a sphingosine moiety. Sphingosine is a class of long-chain compounds with hydroxyl groups on carbons 1 and 3 and an amino group on carbon 2. They form ceramides by joining with fatty acids across the subterminal amino group. GM2 ganglioside accumulates in Tay-Sachs disease, and galactosyl sulfatides accumulate in metachromatic leukodystrophy.
A 32-year-old woman is being evaluated for headaches. They started about 6 months ago and occur a few times per week, lasting until she falls asleep. The pain is constant and focused at the front and back of the head. The pain is unrelated to position and tends to be worse later in the day. There is mild photophobia. Which of the following findings is most likely? a. Slightly reduced neck range of motion and paracervical tenderness b. Papilledema c. Abnormal brain MRI d. Abnormal brain CT e. Abnormal EEG
The answer is a. The history is typical for a tension-type headache. These headaches are often associated with neck muscle spasm leading to reduced neck range of motion and paracervical tenderness. Papilledema and neuroimaging abnormalities would be associated with headaches due to an intracranial mass. The EEG is generally normal in patients with headaches, unless there is underlying damaged brain.
A 52-year-old right-handed woman who has abused intravenous drugs for many years has an HIV antigen test that is positive. CD4+ (helper) T lymphocyte count is normal. A brain CT scan reveals several hemorrhagic lesions. Nerve conduction studies reveal generalized slowing in the legs, and EEG exhibits focal slowing over the left parietal lobe. Car- diac catheterization suggests aortic valve disease, and the patient's CSF is xanthochromic (yellow). Which of the following is the probable site of injury in the CNS? a. An arterial wall b. The ventricular endothelium c. The pia arachnoid d. The dura mater e. The perivenular space
The answer is a. The most likely explanation for this patient's deficits is bleeding from a mycotic aneurysm. This type of aneurysm is usually relatively small and might not be evident on CT scanning or even on arteriography. An arteriogram would miss the lesion if it had destroyed itself when it bled or if the aneurysmal sac was completely thrombosed. The name mycotic is misleading. It suggests a fungal etiology, but it actually refers to the appearance of these aneurysms, which tend to be multiple. These aneurysms occur with either gram-positive or gram-negative infections, but the responsible organisms usually have relatively low virulence. Mycotic aneurysms form over the cerebral convexities with subacute bacterial endocarditis. The aneurysm develops from an infected embolus originating on the diseased heart valves and lodging in the arterial wall. Bleeding from these small aneurysms is largely directed into the subarachnoid space. More virulent organisms that produce valvular heart disease are more likely to produce a meningitis or multifocal brain abscess with seeding of infected emboli to the brain. With acquired immune deficiency syndrome (AIDS), a fungus could be the causative agent, but patients with endocarditis more typically have streptococcal or staphylococcal infections. Even if mycotic aneurysms form with endocarditis, they need not inevitably become symptomatic.
A 52-year-old woman presented to the emergency room with a new onset aphasia. A hemorrhagic left frontal mass is apparent on head CT. The neurosurgical consultant decides to explore the site of the hemorrhage and evacuate the mass that has collected there. He sends tissue from the margin of the blood clot for a frozen section analysis by the pathologist. The tissue is felt to be Kernohan grade IV astrocytoma. Which of the following post- operative therapies is most reasonable? a. Cranial radiotherapy b. Intravenous methotrexate c. Intravenous fludarabine d. Intravenous cyclophosphamide e. Intravenous daunorubicin
The answer is a. There are several different grading schemes for astrocytoma, but Kernohan's classification of grades from I (least malignant) to IV (most malignant) is the one most widely used. Glioblastoma multiforme is an older term for the grade IV astrocytoma and is still in general use. This is a highly malignant tumor that develops most often in the cerebral hemispheres. The most malignant tumors usually exhibit areas of necrosis and have a poor prognosis. Survival with glioblastoma multiforme is usually measured in months rather than years. *Treatment generally consists of gross total resection and radiation therapy*. Survival may be increased to 40 weeks after this combination of therapies, whereas it is on average only 14 weeks after surgery alone. The intravenous medications listed are antineoplastic agents, but they are not effective against this type of tumor. The only chemotherapy generally regarded as useful for this type of primary brain tumor is 1,3-bis (2-chloroethyl)-1- nitrosourea (BCNU), which increases survival only marginally.
A patient undergoes ventriculoperitoneal shunt placement for hydro- cephalus. He is discharged 2 days later, his gait and cognition much improved. The following morning, his wife finds him lying in bed, very confused and complaining of a headache. He is unable to walk. The sur- geon who performed the procedure is concerned that these new symptoms are due to which of the following? a. Chemical meningitis b. Subdural hematoma c. Epidural hematoma d. Seizures e. Bacterial ventriculitis
The answer is b. Up to 28% of patients who undergo ventriculoperitoneal shunting for NPH may suffer major complications, including subdural hematoma. Subdural hematoma occurs because the reduction in intracranial pressure brought on by the reduction in CSF volume may cause the brain to pull away from the covering meninges, stretching and potentially rupturing the bridging veins.
A patient with a meningitis and facial weakness of unknown etiology had been given isoniazid and rifampin. There was no improvement and she is treated with high-dose steroids. Within 1 week of the introduction of prednisone, she develops pain radiating down the back of her right leg and has difficulty dorsiflexing the right foot. This new symptom most likely represents which of the following disorders? a. Borrelia radiculopathy b. Diabetic mononeuritis multiplex c. Isoniazid neuropathy d. Rifampin toxicity e. Tuberculous radiculopathy
The answer is a. This patient has, in effect, developed a noncompressive sciatica, or Lyme neuropathy. Dorsiflexion of the foot is controlled primarily by the anterior tibial muscle. The deep peroneal nerve supplies this muscle and arises from the common peroneal nerve just below the knee. A sciatic nerve injury may also produce foot- drop, and irritation of spinal roots to the sciatic nerve produces the foot- drop that may occur with Lyme radiculopathy. Lyme disease causes painful radiculopathies and peripheral neuropathies. Tuberculosis may cause similar symptoms, but steroid therapy is often useful in suppressing that complaint in chronic tuberculous meningitis. Steroid-induced diabetes mellitus would be unlikely to evolve to the point that a painful neuropathy developed over so short a time.
Which of the following brain structures are currently targets for deep brain stimulation in patients with Parkinson's disease? a. Globus pallidus, medulla, and parietal lobe b. Globus pallidus, subthalamic nucleus, and thalamus c. Hippocampus, medulla, and thalamus d. Medulla, occipital lobe, and subthalamic nucleus e. Parietal lobe, temporal lobe, and thalamus
The answer is b. (Bradley, pp 2138-2139.) Current theory of Parkinson's disease pathology is based on the premise that the substantia nigra pars compacta has decreased dopamine production, which eventually leads to overinhibition of thalamocortical pathways. The thalamus may be directly intervened on to decrease this overinhibition. Alternatively, the globus pallidus interna may be lesioned or stimulated, because it directly inhibits the thalamus. A third approach is to lesion or stimulate the subthalamic nucleus, which has an excitatory connection on the globus pallidus interna and substantia nigra pars reticulata. The medulla, hippocampus, temporal lobe, and occipital lobe are not involved in this pathway.
A 4-year-old previously healthy girl develops an intermittent red, scaly rash over her face, neck, hands, and legs. This is followed by developmental delay, emotional lability, and episodic cerebellar ataxia. She is diagnosed with Hartnup's disease. Her condition may respond to large supplementary doses of which of the following? a. Vitamin C b. Nicotinamide c. Thiamine d. Pyridoxine e. α tocopherol
The answer is b. (Victor, pp 1009-1010.) With Hartnup's disease there is intestinal malabsorption of tryptophan and other neutral amino acids. Tryptophan serves as a precursor for nicotinamide, but with more than 400 mg of nicotinamide daily, the tryptophan malabsorption becomes less problematic. Inheritance appears to be autosomal recessive. Affected children develop a scaly erythematous rash on the face similar to that seen with pellagra. The ataxia exhibited may be episodic.
Hepatosplenomegaly is most likely with which of the following diseases? a. Tay-Sachs disease b. Niemann-Pick disease c. Alpers' disease d. Subacute necrotizing encephalopathy e. Wilson's disease (hepatolenticular degeneration)
The answer is b. (Victor, pp 997-998.) Niemann-Pick disease is inherited as an autosomal recessive trait. By 9 months of age, patients with the infantile form usually have prominent hepatosplenomegaly. A deficiency of sphingomyelinase in hepatocytes is diagnostic for the disease.
A 55-year-old man has a steep decline in his cognitive abilities over a 3-month period. Initial testing is nondiagnostic. He continues to progress and develops myoclonus and a left hemiparesis. Eventually, he dies of an aspiration about 8 months after the onset of symptoms. In the diseases that cause dementia, myoclonus is usually most evident in which of the following? a. Alzheimer's disease b. Creutzfeldt-Jakob disease c. Parkinson's disease d. Huntington's disease e. Pick's disease
The answer is b. *Creutzfeldt-Jakob* disease is a spongiform encephalopathy that produces dementia over the course of months. It is caused by the accumulation within the brain of an abnormal form of a normal protein that resists degradation by proteinases (a proteinase-resistant protein, prion protein, or PrP). *Myoclonic jerks*—abrupt involuntary muscle contractions that may produce brief limb or facial movements—usually appear at some time in the course of this disease. These are often stimulus-sensitive, such that loud noises may provoke them. Similar movements may develop with Huntington's disease, but these patients usually develop more constant and fluid limb movements, called chorea.
An infant has a head CT performed because of a large head and failure to thrive. The diagnosis of hydrocephalus is made. Congenital hydrocephalus may develop as a consequence of which of the following first-trimester maternal disorders? a. Complicated migraine b. Viral infection c. Pseudotumor cerebri d. Chorea gravidarum e. Intervertebral disk herniation
The answer is b. A maternal infection with mumps or rubella virus may produce aqueductal stenosis and, as a consequence, hydrocephalus. The aqueduct of Sylvius connects the third ventricle to the fourth ventricle. The lateral and third ventricles enlarge as the choroid plexus produces fluid that cannot migrate to the subarachnoid space to be reabsorbed.
The parents of a 10-year-old boy bring their child in to see you. The child has been diagnosed with cerebral palsy, and the parents do not really understand what this means. As part of your explanation, which of the fol- lowing would you tell them? a. Cerebral palsy is a static encephalopathy because deficits do not appear after birth b. Cerebral palsy is a static encephalopathy because the injury to the brain does not progress c. Cerebral palsy is a static encephalopathy because affected persons fail to reach any developmental milestones on time d. Cerebral palsy is a static encephalopathy because affected persons have resting tremors e. Cerebral palsy is a static encephalopathy because the EEG exhibits a disorganized background rhythm
The answer is b. A static encephalopathy is one in which brain damage has been arrested but neurologic problems persist. Establishing that the brain lesion is not progressive may require extensive testing. A young child with a static motor disorder is said to have CP. Neurodegenerative diseases with slow or stepwise progressions may appear to be static encephalopathies over the course of months, but prove to be progressive encephalopathies over the course of years. The brain lesion with CP is static, but the deficits associated with CP may evolve as the child matures.
A 45-year-old woman reports to the police her discovery that her husband has added a suspicious material to her food. She has experienced matrimonial problems for several years and has developed progressive fatigue with frequent headache over the prior 3 months. She consulted a physician when she developed recurrent bouts of severe stomach pain and was told by neighbors that she had been talking to herself and attacking invisible assailants. The physician noted that she had an unexplained anemia and white lines running transversely across her fingernails. She also has had problems with her memory, excessive drowsiness, and a sensorimotor neuropathy with absent tendon reflexes. The physician sent a sample of her hair for analysis and found a neurotoxin present.
The answer is b. Acute poisoning with arsenic may cause tonic-clonic seizures or a less dramatic encephalopathy. Hemolysis may be substantial and mucosal irritation may be evident. Death may develop with circulatory collapse if the dose of arsenic is substantial enough. The polyneuropathy that develops with chronic poisoning is resistant to treatment with chelating agents such as BAL. If the patient survives the poisoning, peripheral nerve damage resolves over the course of months or years.
A 43 year-old man presents with a left CN III deficit and headache. Which of the following is the most likely site of the lesion responsible for this presentation? a. Anterior communicating artery b. Posterior communicating artery c. Anterior cerebral artery d. Middle cerebral artery e. Posterior cerebral artery
The answer is b. An aneurysm on the posterior communicating artery is especially likely to compress the oculomotor (third) nerve. Because the pupilloconstrictor fibers lie superficially on this nerve, problems with pupillary activity are routinely early phenomena. An ischemic injury to the third cranial nerve, such as that seen with diabetes mellitus, will usually spare these superficial fibers, presumably because they have a vascular supply that is fairly distinct from that of the rest of the third nerve. The pupillary response to both direct and consensual stimulation will be impaired with compression of these parasympathetic nerve fibers. This means that the pupil in the right eye will not constrict in response to light shining into either the right or the left eye. The normal pupil on the left will constrict with light shining into either the left or the right eye because the sensory input from the right eye is unimpaired. As the aneurysm enlarges, it impinges upon the third-nerve fibers that supply the medial rectus muscle, weakness of which will be responsible for double vision. Lesions of the superior cerebellar artery and posterior cerebral artery can also compress the third nerve, which exits between them. It is therefore important that a complete angiogram, evaluating all four vessels, be performed in the evaluation for subarachnoid hemorrhage and third- nerve palsy.
A 50-year-old man is being evaluated for tinnitus. It is worse on some days than others. Which of the following should he be told may exacerbate the tinnitus? a. Alcohol b. Aspirin c. Glucose d. Diazepam e. Steroids
The answer is b. Aspirin may produce tinnitus in persons usually unaffected by this problem. Patients on high doses of aspirin for rheumatoid arthritis are especially susceptible to this drug- induced tinnitus. Those patients with chronic tinnitus from acoustic trauma or Ménière's disease will find their symptoms worsen with aspirin.
A patient with multiple sclerosis has worsening leg weakness. He has severe spasms of his legs bilaterally, and is increasingly unable to ambu- late because of this. A reasonable symptomatic treatment option would be which of the following? a. Cyclophosphamide b. Baclofen c. Gabapentin d. Amitriptyline hydrochloride e. Propranolol
The answer is b. Baclofen is an antispasmodic agent that may be used in MS. Additional agents that may be used include tiazidine or benzodiazepines. Cyclophosphamide is an immunosuppressive drug that may be used to treat MS, but would not be considered a symptomatic therapy.
A 22-year-old woman presents to the emergency room with an episode of acute painful loss of vision in the right eye. On examination, there is a right afferent pupillary defect and papillitis on funduscopic exam- ination. She has no history of neurologic symptoms. An MRI shows a few foci of T2 signal increase in a periventricular distribution. Which of the fol- lowing is the most appropriate treatment for presumed optic neuritis in this patient? a. Oral prednisone b. Intravenous methylprednisolone c. Cyclophosphamide d. Plasma exchange e. Intravenous gamma globulin
The answer is b. Clinical trials have shown that *intravenous methylprednisolone* for an attack of optic neuritis *reduces the likelihood of developing MS* over 2 years from 16.7% to 7.5%. It also is associated with a better outcome than oral prednisone. Intravenous methylprednisolone is thus recommended by most experts as appropriate therapy for acute exacerbations of MS involving more than sensory manifestations alone.
A young couple comes to your office because of a family history of Friedreich's ataxia. They are in the process of family planning and have several questions regarding the disease. If a patient with Friedreich's ataxia has children, at what stage of life would a child be expected to become symptomatic if the disease was inherited? a. Neonatal period b. Juvenile period c. Early adulthood d. Middle age e. Senescence
The answer is b. Congenital abnormalities are rarely evident with Friedreich's disease, but *kyphoscoliosis, pes cavus, and other musculoskeletal abnormalities may become evident quite early in childhood*. Gait difficulty usually develops during childhood in persons with Friedreich's disease. *Visual loss, syncope, vertigo, and dysarthria may develop during the course of this degenerative disease*, but the appearance of these other problems may be decades after that of the gait ataxia. *Visual loss may develop with optic atrophy or retinitis pigmentosa*. Strictly cerebellar or spinocerebellar tract signs include limb ataxia, nystagmus, dysarthria, and gait difficulty. Systemic problems often found in persons with Friedreich's disease include diabetes mellitus and cardiac conduction defects.
A 6-year-old girl has left facial pain and blurry vision. Careful exam- ination reveals a deficit of the abducens nerve. Which of the following is the most likely etiology? a. Ischemia b. Infection c. Neoplasm d. Trauma e. Hemorrhage
The answer is b. Gradenigo syndrome arises with an osteomyelitis of the petrous pyramid. The abducens and trigeminal nerves are affected as they pass close to the tip of the petrous bone. Chronic ear infections may extend to the petrous pyramid and produce this syndrome if they are not properly managed.
A 65-year-old diabetic woman has aphasia secondary to a stroke involving the inferior division of the left middle cerebral artery. Her hear- ing is intact. Which of the following correctly reflects why dominant tem- poral lobe infarction will not produce complete deafness? a. There is no temporal lobe representation for hearing b. Each cochlear nucleus projects to both temporal lobes c. Deafness results with nondominant hemisphere damage d. Both thalamic and temporal lobe damage must occur e. Both brainstem and temporal lobe damage must occur
The answer is b. Hearing in each ear is repre- sented bilaterally even at the level of the brainstem. Lesions rarely produce sufficient damage in the brainstem to cause unilateral deafness unless they are so massive that the patient is unlikely to be responsive to most stimuli and unlikely to survive. If there is unilateral deafness, the patient should be evaluated to determine whether the hearing loss is conductive or sensorineural.
A 4-year-old boy has the onset of episodes of loss of body tone, with associated falls, as well as generalized tonic-clonic seizures. His cognitive function has been deteriorating. EEG shows 1.5- to 2-Hz spike-and-wave discharges. Which of the following is the most likely diagnosis? a. Landau-Kleffner syndrome b. Lennox-Gastaut syndrome c. Juvenile myoclonic epilepsy d. Mitochondrial encephalomyopathy e. Febrile seizures
The answer is b. Lennox-Gastaut syndrome is characterized by mental dysfunction, multiple seizure types and 1- to 2-Hz generalized spike-wave discharges on EEG. It is often difficult to control the seizures that develop in children with this syndrome. Many affected children have a history of infantile spasms (West syndrome). Infants and children with infantile spasms exhibit paroxysmal flexions of the body, waist, or neck and usually have a profoundly disorganized EEG pattern called hypsarrhythmia.
A 20-year-old ataxic woman with a family history of Friedreich's disease develops polyuria and excessive thirst over the course of a few weeks. She notices that she becomes fatigued easily and has intermittently blurred vision. Which of the following is the most likely explanation for her symptoms? a. Inappropriate antidiuretic hormone b. Diabetes mellitus c. Panhypopituitarism d. Progressive adrenal insufficiency e. Hypothyroidism
The answer is b. More than 10% of patients with Friedreich's disease develop diabetes mellitus. A more life-threatening complication of this degenerative disease is the disturbance of the cardiac conduction system that often develops. Visual problems occur with the hyperglycemia of uncontrolled diabetes mellitus, but even Friedreich's patients without diabetes develop optic atrophy late in the course of the degenerative disease.
A 62-year-old man exhibits excessive sleepiness, slowing of movements, mild depression, and proximal muscle weakness. His proximal limb muscles are obviously atrophied. Although his blood count is normal, routine screening of serum chemistries reveals an elevated calcium level. He also has an elevated serum creatinine with reduced creatinine clearance. The patient has had abdominal discomfort intermittently for several months and has been told that his episodes of joint swelling were due to pseudogout.
The answer is b. Primary hyperparathyroidism develops in the elderly and may be overlooked or misdiagnosed. The elevated calcium (over 11.5 mg/dL) that is characteristic of the disturbance is dismissed as an immobilization phenomenon or misconstrued as evidence of an occult neoplasm. The appearance of pseudogout should raise the probability of hyperparathyroidism substantially. The calcium level may in fact be normal when it is checked, but the parathyroid hormone levels will be elevated.
A 10-year-old girl is referred to a physician because of rapidly deteriorating school performance. Over the course of a few weeks, the child has lost interest in her schoolwork, appeared apathetic at home, and had frequent temper tantrums with little provocation. A psychiatric evaluation reveals that, in addition to emotional lability, the child has substantial intellectual deficits that appear to be new. Within 1 month of this evaluation, the child has a generalized tonic-clonic seizure. A neurologist examining the child discovers chorioretinitis, ataxia, hyperactive reflexes, and bilateral Babinski signs. Her EEG exhibits periodic bursts of high-voltage slow waves followed by recurrent low-voltage stretches (burst suppression pattern). The CSF is remarkable for an increase in the gamma globulin fraction. The child becomes increasingly lethargic and obtunded over the ensuing 2 months. She remains in a coma for several months before dying.
The answer is b. Subacute sclerosing panencephalitis usually develops in children and is rarely seen after the age of 18. Most affected children have had a bout of measles (rubeola) that occurred before they were 2 years old. SSPE may not appear for as long as 6 to 8 years after the episode of measles. Death usually occurs within 1 to 3 years after the onset of symptoms. SSPE produces a CSF pattern similar to that seen with multiple sclerosis, whose features include an increase in the gamma globulin fraction and the presence of oligoclonal bands. The measles virus appears to be directly responsible for this demyelinating disease, and the oligoclonal bands that appear in the CSF include a substantial proportion of measles-specific antibody. Eosinophilic inclusions are typically present in the cytoplasm and nuclei of neurons and glial cells.
The newborn infant with motor neuron disease is likely to exhibit which of the following? a. Seizures b. Hypotonia c. Hypsarrhythmia d. Moro reflexes e. Spina bifida
The answer is b. The child with congenital weakness, hypotonia, and muscle atrophy may have Werdnig- Hoffmann disease, a congenital motor neuron disease. This is an especially lethal form of motor neuron disease and may limit the child's life expectancy to weeks or months. A similar pattern of disease that appears in older children is less lethal and is called Kugelberg-Welander disease. These types of motor neuron diseases are also known as spinal muscular atrophies (SMAs). Anterior horn cell disease is presumed to be a pivotal feature of diseases in this category.
A 70-year-old woman with end-stage renal disease tends to develop restless legs syndrome as she becomes uremic. This may be controlled with which of the following drugs? a. Haloperidol b. Clonazepam c. Caffeine d. Nifedipine e. Rifampin
The answer is b. The restless legs syndrome (Ekbom syndrome) is characterized by a feeling of discomfort in the legs that is relieved by movement. The sensation is felt deep within the limb, and is variably described as a pulling, stretching, or cramping. Restless legs syndrome occurs primarily at night, shortly after the patient lies down. It differs from akathisia, which is a restlessness that occurs during the day- time. It may be associated with peripheral neuropathy and anemia and is seen in patients with chronic renal disease, diabetes mellitus, and many other medical conditions. Exercise before going to bed may alleviate much of the discomfort. Agents that may be effective in alleviating symptoms include clonazepam, gabapentin, L-dopa, and opiates. Neuroleptics, calcium channel blockers, and caffeine may worsen symptoms.
A 51-year-old woman has progressive loss of visual acuity in her left eye. Over the course of 5 years, her acuity has deteriorated from 20/20 to 20/400. An MRI of her brain reveals a large meningioma impinging on the left side of the optic chiasm. There is no associated hydrocephalus. Field Defect?
The answer is b. With a lesion impinging on the chiasm from one side, there should be a field cut in the contralateral field of the contralateral eye. The upper quadrant is preferentially affected. The ipsilateral eye may exhibit little more than an enlarged blind spot that impinges on central vision, a pattern called a centrocecal scotoma. With more substantial damage to the fibers from the eye ipsilateral to the chiasmatic lesion, the patient may have a left nasal hemianopsia, but this rarely appears.
You are working in the emergency room when a 30-year-old man pre- sents with a headache that started yesterday. As he was shoveling snow, he felt a sudden pain in the front of his head. The pain does not throb and has been relatively constant since. He says that now his neck also has become a little stiff. He carries a diagnosis of migraine headaches, but says that this is different than his usual headaches. He is afebrile and has a normal exam except for slight photophobia and mild discomfort with neck flexion. Which of the following is the most appropriate next step in management? a. Obtain a brain MRI b. Obtain a brain CT c. Obtain a cerebral angiogram d. Obtain an EEG e. Obtain a psychiatry consult
The answer is b.This patient gives a good history for subarachnoid hemorrhage. A CT scan will detect blood locally or diffusely in the subarachnoid spaces or within the brain or ventricular system in more than 90% of cases. It is more sensitive than an MRI in most cases and can be obtained more quickly. A cerebral angiogram could diag- nose the etiology of a subarachnoid hemorrhage, such as an aneurysm. However, it is an invasive test and should not be done without first attempting to confirm the diagnosis with less risky tests. If you suspect seizures, an electroencephalogram might be useful. As this patient did not have any loss of consciousness, personality changes, hallucinations, or rhythmic movements suggestive of seizures, there is no indication for a psychiatry consult.
A27-year-old man develops recurrent episodes of involuntary movements. He abused intravenous drugs for several years and has had several admissions for recurrent infections, including subacute bacterial endocarditis. His involuntary movements are largely restricted to the right side of his body and are associated with hoarseness and difficulty swallowing. The patient has lost 40 lb over the past 4 months. Examination reveals diffuse lymphadenopathy and right-sided hypertonia. His CSF is normal except for a slight increase in protein content. Computed tomography reveals a large area of decreased density on the left side of the cerebrum. EEG reveals diffuse slowing over the left side of the head. Biopsy of this lesion reveals oligodendrocytes with abnormally large nuclei that contain darkly staining inclusions. There is extensive demyelination and there are giant astrocytes in the lesion. Over the course of 1 month, the man exhibits increasing ataxia. Within 2 months, he shows evidence of mild dementia and seizures. Within 3 months of presentation, his dementia is profound and he has bladder and bowel incontinence. Over the course of a few days, he becomes obtunded and dies.
The answer is c. (Bradley, pp 1594-1596.) This patient probably had AIDS with PML as a complication of that disease. The inclusion bodies in the oligodendrocyte nuclei are JC virus, a papillomavirus. Primary infection with JC virus is universal and asymptomatic. Immunosuppression leads to reactivation of the virus. Diagnosis is typically made by MRI, which shows multiple focal well-defined white matter lesions that do not enhance or have mass effect. Cerebrospinal fluid PCR for JC virus is also available, obviating the need for brain biopsy in most cases. Treatment with cytarabine arabinoside has not been shown to be effective in clinical trials. Less than 10% of patients may experience spontaneous remission. PML may also develop with lymphomas, leukemias, or sarcoid, but the inci- dence of this disease in the U.S. population has expanded greatly since the dissemination of HIV in the population.
A 26-year-old man develops hemoptysis and dyspnea over the course of 3 months. His physician suspects tuberculosis and starts him on triple therapy with isoniazid (isonicotinic acid hydrazide), rifampin, and ethambutol. After 1 month of treatment, the patient's liver enzymes show slight elevations, but the treatment is continued. The hemoptysis stops by 2 months, but the patient complains of pins-and-needles sensations in his feet. Neurologic examination reveals hypoactive deep tendon reflexes in the legs and slightly impaired position sense. Strength is good in all limbs. Dx?
The answer is c. Any patient treated with isoniazid must receive supplemental pyridoxine. Isoniazid does not interfere with pyridoxine absorption, but it does interfere with its participation in metabolic pathways. Persistently low pyridoxine activity leads to the development of a peripheral neuropathy. This is most likely to be seen as an isolated deficiency in patients on antituberculous therapy.
A 17-year-old girl develops a painful vesicular rash around her left eye. This is followed by blurry vision that occurs only when both eyes are open. She is diagnosed with varicella zoster ophthalmicus. Which ocular motor nerve is most likely to be affected? a. Superior division of the third b. Inferior division of the third c. Fourth (trochlear) d. Sixth (abducens) e. Long ciliary
The answer is c. (Victor, p 287.) Varicella zoster, previously known as herpes zoster, spreads to the face along the trigeminal nerve. The fourth nerve is presumably involved because it shares its nerve sheath with the ophthalmic division of the trigeminal nerve. The third and sixth nerves may also be involved with varicella zoster, but this occurs much less frequently than involvement of the fourth nerve.
A 28-year-old man presents with right eye pain and blurry vision developing over 3 days. After examination and further history, a diagnosis of papillitis is made. How can papillitis be distinguished from the papilledema of increased intracranial pressure? a. Degree of swelling of the optic disc b. Associated homonymous hemianopsia c. Characteristic visual loss d. Associated limitation of eye movement e. Loss of red reflex
The answer is c. (Victor, pp 261-262.) Visual loss is usually substantial with papillitis, an inflammation of the optic nerve head, and inconsequential with papilledema. Patients with papillitis usually also have pain on moving the globe and sensitivity to light pressure on the globe. About 1 in 10 patients have both eyes involved simultaneously. Papillitis is often an early sign of multiple sclerosis.
A 35-year-old woman presents with slowly evolving left arm ataxia, left-sided head tilt, dysarthria, and left facial weakness. The patient denies vertigo, tinnitus, or hearing loss. MRI reveals a posterior fossa mass that lies close to the bone and enhances with contrast. Which of the following is the most likely explanation for this lesion? a. Cerebellar infarction b. Cerebellar hemorrhage c. Meningioma d. Schwannoma e. Astrocytoma
The answer is c. (Victor, pp 692-693.) Any type of stroke in the cere- bellum would be expected to evolve over the course of hours, rather than days or weeks. With signs and symptoms that evolve slowly, a neoplasm is more likely. Because there was no involvement of the eighth cranial nerve, the most probable neoplasm is a meningioma. This tumor also appears to arise from bone, another indication that it is most likely a meningioma.
A 7-year-old boy is taken by his parents to see a dermatologist. They have noticed nodules on his face and are concerned. The dermatologist tells them that their child has adenoma sebaceum. Adenoma sebaceum of the face is especially common with which of the following diseases? a. Neurofibromatosis b. Sturge-Weber syndrome c. Tuberous sclerosis d. Ataxia telangiectasia e. Fragile X syndrome
The answer is c. Adenoma sebaceum occurs in about 90% of patients with tuberous sclerosis. A depigmented lesion, called a shagreen patch, occurs in only about 20% of these patients. Adenoma sebaceum usually becomes apparent over the malar eminences of the face between 2 and 5 years of age and may evolve into difficult-to-treat angiofibromas of the skin. In ataxia telangiectasia, facial telangiectasias may develop. Sturge-Weber syndrome is characteristically associated with a port-wine spot over part of the face. Patients with neurofibromatosis often have café au lait spots, but these do not usually occur on the face.
A 64-year-old man presents to the emergency room with convulsive seizures. A precontrast CT of the brain reveals a hemorrhagic mass in the left frontal lobe, but there is little apparent shift of brain structures and no ventricular enlargement. Two hours after the patient's seizures have stopped, his blood pressure is still elevated at 180/100 mmHg and his pulse is slow at 50/min. Although the patient is still unconscious, he appears to have decreased tone on the right side of his body. The physician should request which of the following interventions? a. Intravenous clonidine (Catapres) to lower the blood pressure b. Placement of a cardiac pacemaker to manage the bradyarrhythmia c. Neurosurgical consult d. Placement of a ventriculoperitoneal shunt e. Intravenous tissue plasminogen activator (TPA)
The answer is c. An expanding intracranial mass will produce an elevated blood pressure and a slow heart rate. This is called the Cushing effect. This man may have a neoplasm in the brain or amyloid bleed. The site of the hemorrhage is unlikely with chronic hypertension or aneurysm. A biopsy of the mass would help identify the underlying lesion, although it is not urgent. Metastatic neoplastic disease is a possibility but is less likely than a glioblastoma multiforme at this age. The administration of TPA is contraindicated because this drug will increase the risk of rebleeding. Placement of a drain is not suggested by the clinical picture because there was no evidence of obstruction to the flow of CSF.
Glaucoma develops in nearly one-third of children with which of the following? a. Type 1 neurofibromatosis b. Type 2 neurofibromatosis c. Sturge-Weber syndrome (encephalotrigeminal angiomatosis) d. Tuberous sclerosis e. Arnold-Chiari malformation
The answer is c. Children with Sturge-Weber syndrome have large port-wine spots on their faces, contralateral hemiparesis, retardation, and seizures, as well as glaucoma. Skull radiographs reveal intracranial calcifications that are associated with leptomeningeal angiomatosis. This syndrome results from a defect on chromosome 3.
A 45-year-old man has noticed over the past 6 months that his sense of smell is not as sensitive as it used to be. On examination he has unilateral anosmia, ipsilateral optic atrophy, and contralateral papilledema. Which of the following is the most likely diagnosis? a. Pseudotumor cerebri b. Multiple sclerosis (MS) c. Olfactory groove meningioma d. Craniopharyngioma e. Nasopharyngeal carcinoma
The answer is c. Ipsilateral optic atrophy and contralateral papilledema in association with an intracranial tumor constitute the Foster-Kennedy syndrome. A meningioma of the olfactory groove may produce this syndrome if it extends posteriorly to involve the ipsilateral optic nerve. Compression on the optic nerve by the tumor produces atrophy and interferes with transmission of the increased intracranial pressure down the optic sheath. The increased intracranial pressure is reflected in the papilledema apparent in the contralateral eye.
A 45-year-old right-handed man who has been HIV positive for the past 3 years has noticed some sort of visual change over the past 1 to 2 months. It is difficult for him to describe, but it is some sort of distortion of part of his right visual field. There is a 4-cm rim-enhancing lesion in the left occipital lobe that is revealed by MRI. Which of the following tumor types is common in the brain of patients with AIDS, but otherwise extremely rare? a. Lymphocytic leukemia b. Metastatic lymphoma c. Primary lymphoma d. Kaposi's sarcoma e. Lymphosarcoma
The answer is c. Kaposi's sarcoma is unusually common in patients with AIDS, but it is rarely metastatic to the brain. Metastatic lymphomas producing meningeal lymphomatosis are not especially rare in the general population, but primary lymphomas (i.e., lymphomas apparently arising in the CNS) were rare before the AIDS epidemic. The primary brain lymphoma usually presents as a solitary mass and can occur anywhere in the brain, but it does have a predilection for the periventricular structures.
A 36-year-old man abruptly loses vision in one eye. His retina appears cloudy and grayish yellow with narrowed arterioles. The fovea appears cherry red, and the vessels that are obvious appear to have segmented columns of blood. Which of the following is the most likely diagnosis? a. Chorioretinitis b. Occlusion of the central retinal vein c. Occlusion of the central retinal artery d. Optic neuritis e. Tay-Sachs disease
The answer is c. Occlusion of the central retinal artery may be due to atheromatous particles, fibrin-platelet emboli, or local retinal artery compression. The visual loss is usually *painless and irreversible*. Occlusion of the internal carotid artery—the artery from which the ophthalmic and ultimately the retinal arteries originate—need not produce ischemic damage to the retina if collateral supply to the retinal artery is sufficient.
A 29-year-old woman has progressive gait disorder and dysmetria. Laboratory studies include a hematocrit of 55% and a routine urinalysis, which reveals excess protein and some RBCs in the urine. Urine culture is negative. The initial physical examination reveals an enlarged liver and spleen. Additional physical findings will most likely include which of the following? a. A Kayser-Fleischer ring around the cornea b. Hypopigmented (ash-leaf) spots on the trunk c. Telangiectasias in the fundi on retinal examination d. Bilateral hearing loss e. Generalized hyporeflexia
The answer is c. The association of erythrocytosis with cerebellar signs, microscopic hematuria, and hepatospleno- megaly suggests *von Hippel-Lindau syndrome*. This hereditary disorder is characterized by *polycystic liver disease, polycystic kidney disease, retinal angiomas (telangiectasia), and cerebellar tumors*. This is an autosomal dominant inherited disorder with variable penetrance. Men are more commonly affected than women. Although neoplastic cysts may develop in the cerebellum in persons with von Hippel-Lindau syndrome, these usually do not become sufficiently large to cause an obstructive hydrocephalus. Other abnormalities that occur with this syndrome include adenomas in many organs. Hemangiomas may be evident in the bones, adrenals, and ovaries. Hemangioblastomas may develop in the spinal cord or brainstem, as well as in the cerebellum. This syndrome is not associated with acoustic schwannomas that could cause bilateral hearing loss, and it is not accompanied by peripheral neuropathy, which could cause diffuse hyporeflexia.
A 19-year-old man is hit in the face with a leadpipe. The ocularmotor muscle most likely to be injured in this case is that innervated by which of the following? a. Superior division of the third cranial nerve b. Inferior division of the third cranial nerve c. Fourth (trochlear) cranial nerve d. Sixth (abducens) cranial nerve e. Long ciliary nerve
The answer is c. The fourth cranial nerve innervates the superior oblique muscle. Because this muscle extends far anterior in the orbit, it is at high risk of injury with trauma to the orbit or the full face. The third nerve is especially vulnerable to pressure from aneurysms, but it is usually not disturbed with head trauma unless there are local fractures impinging on it. Injury to the fourth nerve with facial trauma will usually induce a slight head tilt to compensate for impaired intorsion of the affected eye.
A 75-year-old man has malaise and slowly progressive weight loss for the better part of 3 months. Laboratory tests reveal a hematocrit of 32%, an erythrocyte sedimentation rate (ESR) of 97 mm/h, and a white blood cell (WBC) count of 10,700 cells per μL. Serum CPK and thyroxine (T4) levels are normal. Which of the following is the most likely explanation for the patient's complaints? a. Polymyositis b. Dermatomyositis c. Polymyalgia rheumatica d. Rheumatoid arthritis e. Hyperthyroid myopathy
The answer is c. The markedly elevated sedimentation rate, anemia, weight loss, and malaise in a person of this age suggest polymyalgia rheumatica, although the same complaints in someone 20 years younger could not be explained on the basis of this disorder. Fever may also be evident in the affected person. This constellation of symptoms also suggests an occult neoplasm or infection, and investigations should be conducted to reduce the likelihood of overlooking one of these diseases. Polymyalgia rheumatica is an arteritis of the elderly and is improbable in someone less than 60 years of age. The normal CPK activity markedly reduces the likelihood that this myalgia is the result of polymyositis or dermatomyositis. The new onset of rheumatoid arthritis at this age is also improbable. A hyperthyroid myopathy in the face of a normal T4 level is possible on the basis of an elevated T3 level, but it is also much less likely than polymyalgia rheumatica in this age group.
A 42-year-old man notices that his right pupil is smaller than the left. His wife has also commented that the right eye is "droopy looking." The only remarkable recent history is that he was tackled a little hard while playing football the day before. Magnetic resonance imaging (MRI) of the head and neck in this patient would be expected to show which of the fol- lowing? a. Increased T2 signal in a periventricular distribution b. Contrast enhancement along the tentorial margin c. Increased T1 signal in the wall of the left carotid artery d. Enlarged optic nerve in the orbit e. Thrombosed cavernous sinus aneurysm
The answer is c. The most likely cause of this patient's symptoms and signs is a *carotid dissection*, caused by a tear in the intimal lining of the vessel with penetration of blood beneath the intimal surface, forming an intramural hematoma. This may be seen as an area of increased signal within the lining of the vessel on T1-weighted images, on which a blood clot appears bright. Typically, this takes a crescentic pattern, and the lumen of the vessel is displaced eccentrically. An increased T2 signal in a periventricular distribution is typical of the plaques of demyelinating disease seen in multiple sclerosis. Contrast enhancement along the tentorial margin might be seen in inflammatory diseases of the dura mater or meninges. An enlarged optic nerve might be seen in the setting of optic neuritis or infiltration of the nerve by a tumor or other process, and this would be expected to cause visual impairment. A cavernous sinus aneurysm that is sufficient to produce oculosympathetic palsy would be expected to cause other deficits in ocular motility, because the cranial nerves subserving oculomotor function course through the cavernous sinus.
A 50-year-old man had a brainstem stroke following a vertebral artery dissection secondary to an acute sports-related injury. This patient might be expected to develop dysphagia secondary to involvement of which of the following structures? a. Nucleus solitarius b. Nucleus and descending tract of CN V c. Nucleus ambiguus d. Lateral spinothalamic tract e. Inferior cerebellar peduncle
The answer is c. The nucleus ambiguus, located in the ventrolateral medulla, *contains the motor neurons that contribute to the ninth (glossopharyngeal) and tenth (vagus) cranial nerves*. The motor neurons of the nucleus ambiguus innervate the striated muscles of the larynx and pharynx as well as provide the preganglionic parasympathetic supply to thoracic organs, including the esophagus, heart, and lungs. Injury to this nucleus and its pathways causes hoarseness and dysphagia.
A child with rapid downward deviation of both eyes followed by slow upward conjugate eye movements probably has which of the following? a. SSPE b. MS c. Pontine glioma d. Cervicomedullary junction ischemia e. Cerebral palsy (CP)
The answer is c. The phenomenon described is com- monly referred to as ocular bobbing. It is an involuntary movement that usu- ally develops with pontine damage. Damage to the cerebellum occasionally produces a similar disturbance of eye movements
A 17 year-old woman has weakness of left shoulder abduction and elbow flexion, with good strength in hand and forearm muscles. Which of the following is most likely to cause an injury limited to the upper brachial plexus? a. Node dissections in the axilla b. Pancoast tumor c. Birth trauma d. Dislocation of the head of the humerus e. Aneurysm of the subclavian artery
The answer is c. The upper brachial plexus includes the fifth and sixth cervical spinal roots. Damage to these roots occurs during a difficult birth if the head and the shoulder are forced widely apart. This could result in stretching or even avulsion of these cervical spinal roots from the spinal cord. Node dissection in the axilla or a Pancoast tumor at the apex of the lung may produce brachial plexus injury, but it is the lower plexus that is vulnerable. Dislocation of the head of the humerus or aneurysm of the subclavian artery will typically injure the cords of the brachial plexus, the final elements of the plexus from which the principal nerves of the arm arise.
A 58-year-old man with type 2 diabetes presents with the acute onset of double vision. Examination reveals a deficit of the third cranial nerve. A third-nerve palsy associated with diabetes mellitus is usually characterized by which of the following? a. Poor pupillodilation b. Poor pupilloconstriction c. Sparing of pupillary function d. Inversion of the affected eye e. Upward deviation of the affected eye
The answer is c. The vessel usually obstructed with diabetic third-nerve injury is deep in the third nerve. The superficial fibers to the iris are supplied by a separate set of vessels, and these are usually spared with diabetes mellitus. With the damaged third nerve, the affected person may complain of pain in and about the eye.
A 64-year-old man who has had hypertension for more than 30 years is being examined. The most obvious changes seen during retinal exam would include which of the following? a. Retinal tears b. Optic atrophy c. Segmental narrowing of arterioles d. Drusen e. Telangiectasias
The answer is c. The vessels apparent on funduscopic examination of the retina are arterioles and venules. In addition to segmental narrowing of arterioles, the retina may exhibit arteriolar straightening and arteriolar-venular compression. The thickened arteriolar wall compresses the venule at the point where they cross, a pattern often referred to as nicking.
Routine funduscopic examination of a 52-year-old man reveals small, discrete red dots located in largest numbers in the paracentral region. Such retinal microaneurysms most often occur with which of the following? a. Sarcoidosis b. Chronic hypertension c. Diabetes mellitus d. Anterior communicating aneurysms e. Chorioretinitis
The answer is c. These aneurysms appear as small red dots on the surface of the retina. *They may appear as one of the first manifestations of diabetes mellitus* and are rarely larger than 90 μm across. They may be more obvious in green light. A proliferative retinopathy may occur along with these microaneurysms in the patient with diabetes mellitus.
A 26-year-old recent immigrant from Brazil presents to the hospital with a subacute, worsening paraparesis. The patient had worked in the lum- bar industry deep in the Amazon jungle. MRI of the spinal cord is abnormal and a biopsy reveals widespread granulomas. In the midst of one granu- loma is an ovoid mass with a spine extending from one side. The patholo- gist interprets this as a parasitic ovum. If the pathologist is correct, which of the following is the most likely cause of the lesion? a. Taenia solium b. Entamoeba histolytica c. Schistosoma mansoni d. Schistosoma japonicum e. Treponema pallidum
The answer is c.T. pallidum may produce a granulomatous lesion (gumma) in the spinal cord, but this young man has an ovum in the granuloma, which suggests the much more common transverse myelitis attributable to schistosomiasis. Both S. mansoni and S. japon- icum embolize eggs to the CNS, but it is S. mansoni that is endemic in Puerto Rico and in locations in South America and that embolizes to the lumbar spinal cord. This patient should be treated with an antischistosomal agent such as praziquantel. Even with treatment, the reversal of disability produced by this spinal cord injury is usually negligible.
A 4-year-old boy develops progressive gait ataxia and limb weakness over the course of 3 months. Neurologic assessment reveals diffusely absent deep tendon reflexes, proximal muscle weakness, ophthalmoparesis, and poor pain perception in the feet. Blood tests reveal elevated creatine phosphokinase (CK) levels and abnormally high serum bilirubin levels. Further investigations of hepatic function reveal that the child has a cholestatic
The answer is d. (Victor, p 1224.) Vitamin E deficiency that causes neurologic disease is rare, but when it does develop it is usually during early childhood. The most common syndrome involves spinocerebellar degeneration, polyneuropathy, and pigmentary retinopathy. Clarke's columns, the spinocerebellar tracts, the posterior columns, the nuclei of Goll and Burdach, and sensory roots are especially likely to exhibit degeneration in persons with vitamin E deficiency. The most obvious symptom of the deficiency is likely to be ataxia. hepatobiliary disorder, but there is no evidence of hepatic dysfunction sufficient to cause an encephalopathy.
A 26-year-old heroin addict has been using a street version of artificial heroin. The drug actually contains 1-methyl-4-phenyl-1,2,3,6- tetrahydropyridine (MPTP). The neurological syndrome for which he is at risk is clinically indistinguishable from which of the following? a. Huntington's disease b. Friedreich's disease c. Sydenham's chorea d. Parkinson's disease e. Amyotrophic lateral sclerosis
The answer is d. (Victor, p 73.) Young adults who have self- administered MPTP in an effort to achieve an opiate high have developed progressive damage to the substantia nigra. The neurologic syndrome that results from this damage is indistinguishable from Parkinson's disease, except that it evolves over weeks or months rather than years. Affected per- sons exhibit rigidity, tremor, and bradykinesia. That a toxin can produce a syndrome indistinguishable from Parkinson's disease has increased specu- lation that some—perhaps many—persons with Parkinson's disease have had environmental exposure to a toxin that produced degeneration of the substantia nigra.
A 60-year-old right-handed man presents with visual loss. About 2 weeks before, he began to notice difficulty seeing the television. Within 1 week, he noticed that the inferior field of vision in the right eye was much worse than the top of his vision. Within a few more days, he noticed the bottom of the vision in his left eye worsen as well. This has been painless. He has otherwise felt well, without headaches or cognitive changes. An ophthalmologist saw bilateral papillitis with white exudates of the nasal part of the discs. There is no history of alcohol use, and the patient has stopped smoking since his heart transplant. On examination, he appears well. Blood pressure is 160/80; pulse is 100 and regular. There are no carotid bruits. Pupils are equal and reactive. Visual acuity is 20/400 OU, with central-inferior scotomas (left larger than right). Neurologic exam is otherwise normal. An MRI scan with and without gadolinium contrast agent, including orbital cuts, is negative, as is CSF examination. This patient's history, exam, and laboratory findings are now most consistent with which of the following diagnoses? a. Cyclosporine toxicity b. Occipital lobe lymphoma c. Tobacco-alcohol amblyopia d. Ischemic optic neuropathies e. Retinal detachment
The answer is d. *Ischemic optic neuropathy, often called anterior ischemic optic neuropathy (AION)*, is the most common cause of acute monocular blindness. This condition presents as sudden, painless loss of vision in one eye. Symptoms may progress over several days, and the visual loss is permanent. The visual field defect is typically an inferior altitudinal defect, with involvement of central vision and a consequent loss of acuity. In up to one-third of patients, the opposite eye may become involved soon afterward. Hypertension and diabetes mellitus appear to be risk factors, as for most small-vessel disease. The responsible arterial occlusion is of the posterior ciliary artery, a branch of the ophthalmic artery, which supplies the optic nerve. Typically, this condition is not associated with carotid artery disease. Giant cell arteritis (temporal arteritis) needs to be excluded, because it can be treated with steroids.
A 39-year-old woman was involved in a head-on collision at approxi- mately 40 miles per hour. She was wearing her seat belt, but still sustained a cervical cord injury from hyperflexion and extension. A cervical syrinx is most likely to evolve in this patient if there has been which of the following? a. Intraspinal hyperthermia b. Intraspinal hypothermia c. Intraspinal transient ischemia d. Intraspinal contusion e. Intraspinal demyelination
The answer is d. After cervical cord contusion, cyst formation may occur as damaged tissue is removed. This is especially likely if there has been extensive intraspinal hemorrhage. Ischemic damage may produce similar changes, but the ischemia must be substantial and persistent enough to produce infarction of spinal cord tissue. Demyelination does not lead to syringomyelia, even in cases with extensive intraspinal demyelination.
A child is born to a 19-year-old woman who has had two to eight drinks per day throughout her pregnancy. What is the major pathologic effect of alcohol on the central nervous system of the developing fetus? a. Cerebral ischemia b. Periventricular hemorrhage c. Macrocephaly d. Impaired neuronal migration e. Holoprosencephaly
The answer is d. Alcohol abuse in pregnant women is associated with three major kinds of abnormalities in the developing fetus: intrauterine and postpartum growth retardation, dysmorphic facies in the newborn, and effects on the development of the CNS. The broad range of neurologic and systemic abnormalities observed in children born to alcohol-abusing women is referred to as the fetal alcohol syndrome. Alcohol is teratogenic at high doses and may interfere measurably in fetal development with exposure at any dose. Although the mechanism of alcohol's effect on the developing brain is not entirely clear, *it appears that alcohol acts primarily to impair neuronal migration*. This may result in formation of heterotopias (collections of cortical neurons in abnormal locations), cortical disorganization, and malformations of the cerebellum and brainstem. Mental retardation, learning disabilities, hyperactivity, and microcephaly, not macrocephaly, are the common clinical neurologic consequences of fetal alcohol syndrome. Ischemia and hemorrhagic complications are not part of the syndrome. Holoprosencephaly refers to a failure of the two sides of the frontal cerebrum to separate properly, leading to a fusion of the frontal poles and hippocampi with no interhemispheric fissure.
A 75-year-old man with a history of recent memory impairment is admitted with headache, confusion, and a left homonymous hemianopsia. He has recently had two episodes of brief unresponsiveness. There is no history of hypertension. Computed tomography (CT) scan shows a right occipital lobe hemorrhage with some subarachnoid extension of the blood. An MRI scan with gradient echo (susceptibility) sequences reveals foci of hemosiderin in the right temporal and left frontal cortex. Which of the fol- lowing is the most likely cause of this patient's symptoms and signs? a. Gliomatosis cerebri b. Multi-infarct dementia c. Mycotic aneurysm d. Amyloid angiopathy e. Undiagnosed hypertension
The answer is d. Cerebral amyloid angiopathy (CAA), or congophilic angiopathy, is the *most common cause of lobar hemorrhage in elderly patients without hypertension*. The deposition of β-amyloid protein (the same as that found in Alzheimer's disease) in brain blood vessels leads to disruption of the vessel walls, which predisposes them to hemorrhage. Patients are usually over age 70 and may present with multiple cortical hemorrhages, with or without a history of dementia. At times, additional hemorrhages may be seen only on special imaging techniques, such as gradient echo MRI, which magnifies the effects of hemosiderin in regions of prior hemorrhage.
The most prominent areas of degeneration with Friedreich's disease are in which of the following areas? a. Cerebellar cortex b. Inferior olivary nuclei c. Anterior horns of the spinal cord d. Spinocerebellar tracts e. Spinothalamic tracts
The answer is d. Degeneration occurs primarily in the spinal cord rather than the cerebellum or brainstem in patients with Friedreich's disease. Both the dorsal and ventral spinocerebellar tracts are involved. The other spinal cord structures exhibiting degeneration include the posterior columns and the lateral corticospinal tracts.
A boy has the onset of difficulty walking at 16 months. Reflexes are decreased. Over the course of several months, the patient becomes dys- arthric and mental functioning decreases. Testing reveals that the patient has a deficiency of arylsulfatase A. Which of the following is the most likely diagnosis? a. Sandhoff's disease b. Tay-Sachs disease c. Gaucher's disease d. Metachromatic leukodystrophy e. McArdle's disease
The answer is d. Hexosaminidase deficiencies produce Sandhoff's and Tay-Sachs diseases. Glucocerebrosidase is deficient in Gaucher's disease. Phosphofructokinase deficiency is usually symptomatic as a disturbance of skeletal muscle function. The enzymatic defect in metachromatic leukodystrophy is transmitted in an autosomal recessive fashion. The affected person usually has retardation, ataxia, spasticity, and sensory disturbances, but individual elements of this disorder may appear alone in less serious cases. The disease is usually symptomatic during infancy.
Which of the following treatments should be recommended to improve the outcome of a patient with a traumatic head injury? a. Corticosteroids b. Prophylactic hyperventilation c. Hyperthermia d. Hypothermia e. Prophylactic anticonvulsants
The answer is d. Hypothermia has been shown to reduce cerebral injury from ischemia both in experimental models and in clinical studies of patients with traumatic brain injury. *Hypothermia decreases cerebral metabolism, reduces acidosis, attenuates changes in the blood-brain barrier, and inhibits the release of excitatory neurotransmitters that can be harmful*. Corticosteroids, prophylactic hyperventilation, and prophylactic anticonvulsants have not been shown to be of benefit in the long-term prognosis of severely head-injured patients. Hyperthermia is detrimental to such patients.
A patient has had progressive, chronic liver failure for the past 5 years. At the time of death, he would be expected to exhibit changes in which type of brain cells? a. Oligodendrocytes b. Striatal neurons c. Pigmented cells of the substantia nigra d. Astrocytes e. Inferior olivary neurons
The answer is d. Long-standing hepatic disease may produce a profound encephalopathy, but changes in the brain are notably sparse with portal-systemic encephalopathy. The most obvious change is an increase in Alzheimer's type II astrocytes. These astrocytes are relatively large cells. Rare patients show more dramatic changes, which include neuronal loss and focal necrosis. With chronic alcoholism and hepatic insufficiency, patients exhibit a loss of Purkinje cells in the cerebellum, but this is a consequence of alcohol toxicity or thiamine deficiency rather than of toxic injury from the hepatic dysfunction.
A 27-year-old man begins to experience infrequent episodes of nau- sea, warmth rising through his body, and an unusual odor like rotting fish. His girlfriend notices that afterward he may develop twitching of the left side of his face and an inability to speak for several minutes. Afterward the man appears dazed and cannot remember what has occurred. He has other- wise been well. Magnetic resonance imaging (MRI) of his brain is most likely to show a lesion in which of the following areas? a. Left occipital lobe b. Right frontal lobe c. Cribriform plate d. Uncus e. Left parietal lobe
The answer is d. Many patients with complex partial seizures have a preseizure phenomenon (the aura) that alerts them to an impending seizure. This patient's aura includes an olfactory hallucination, which is usually associated with lesions of the *mesial temporal lobe, particularly the uncus or parahippocampal gyrus*. Diseases that can affect that region include tumors, trauma, and mesial temporal sclerosis.
A man working in a poorly regulated felt processing plant develops tremors and memory disturbances over the course of months. He seeks medical help when tremors of his tongue and lips became embarrassing and he is injured during a fall. His family notes progressive irritability and depression. On neurologic examination, he has prominent gait ataxia, limb and facial tremors, and decreased pain and temperature sense in his feet. Toxin?
The answer is d. Persons who cured felt (used in the manufacture of hats) with nitrate of mercury often developed *pronounced personality changes, tremor, and ataxia*. This type of poisoning is now more typically seen in paper, pulp, and electrochemical plants that use phenyl mercury as part of the manufacturing process. Pathologic changes in the CNS are usually prominent in the cerebellum and include *extensive damage to the granular cell layer of the cerebellum*. The calcarine cortex of the occipital lobe is also especially vulnerable, and damage to this tissue correlates with constriction of the visual fields.
Language testing is most likely to uncover which of the following deficits in a patient with Alzheimer's disease? a. No abnormalities b. Mutism c. Conduction aphasia d. Transcortical sensory aphasia e. Transcortical global aphasia
The answer is d. The major clinical features of Alzheimer's disease are memory impairment, aphasia, apraxia, and neuropsychiatric impairment, including mood disturbances, delusions and hallucinations, personality changes, and behavior disturbances. The language disturbance may take the form of decreased fluency, dysnomia, and transcortical sensory aphasia, which refers to a reduction in the ability to understand complex linguistic structures. Repetition of verbal material is intact.
A 62-year-old right-handed man has "involuntary twitches" of his left hand. He first noticed between 6 months and 1 year ago that when he is at rest, his left hand shakes. He can stop the shaking by looking at his hand and concentrating. The shaking does not impair his activities in any way. He has no trouble holding a glass of water. There is no tremor in his right hand and the lower extremities are not affected. He has had no trouble walking. There have been no behavioral or language changes. On examina- tion, a left hand tremor is evident when he is distracted. Handwriting is mildly tremulous. He is very mildly bradykinetic on the left. The most likely exam finding would be which of the following? a. Upper motor neuron pattern of weakness on the left b. Lower motor neuron pattern of weakness on the left c. Bilateral upper motor neuron pattern of weakness d. Mild cogwheel rigidity on the left only with distraction e. Bilateral severe cogwheel rigidity
The answer is d. This patient gives a typical history for early Parkinson's disease. The classic triad is asymmetric resting tremor, rigidity, and bradykinesia. The rigidity is generally severe later, not early in the disease. Parkinson's disease is not characterized by weakness.
A 16-year old girl with complex partial seizures and mild mental retardation has an area of deep red discoloration (port-wine nevus) extending over her forehead and left upper eyelid. A CT scan of her brain would be likely to reveal which of the following? a. A hemangioblastoma b. A Charcot-Bouchard aneurysm c. An arteriovenous malformation d. A leptomeningeal angioma e. A fusiform aneurysm
The answer is d. This patient has encephalofacial angiomatosis (Sturge-Weber syndrome), a congenital disturbance that produces facial cutaneous angiomas with a distinctive and easily recognized appearance, along with *intracranial abnormalities such as leptomeningeal angiomas*. Persons with the syndrome may be mentally retarded and often exhibit hemiparesis or hemiatrophy on the side of the body opposite the port-wine nevus. Both men and women may be affected, and seizures may develop in affected persons. The nevus associated with Sturge-Weber syndrome usually extends over the sensory distribution of the first division of the trigeminal nerve. The lesion usually stays to one side of the face. Affected persons will usually also have an angioma of the choroid of the eye. Intracranial angioma is unlikely if the nevus does not involve the upper face. Deficits develop as the person matures and may be a consequence of focal ischemia in the cerebral cortex that underlies the leptomeningeal angioma. Hemangioblastomas are vascular tumors seen in association with polycystic disease of the kidney and telangiectasias of the retina (von Hippel-Lindau syndrome). Charcot-Bouchard aneurysms are very small and may be microscopic. They develop in patients with chronic hypertension and most commonly appear in perforating arteries of the brain. The lenticulostriate arteries are most commonly affected. Hemorrhage from these aneurysms is likely, and the putamen is the most common site for hematoma formation. Hemorrhage may extend into the ventricles and lead to subarachnoid blood. Other locations commonly affected include the caudate nucleus, thalamus, pons, and cerebellum. The dentate nucleus of the cerebellum is especially susceptible to the formation of Charcot-Bouchard aneurysms. Fusiform aneurysms are diffusely widened arteries with evaginations along the walls, but without stalks as occur with the typical berry-shaped structures of the saccular aneurysm. This type of aneurysm may be a late consequence of arteriosclerotic damage to the artery wall.
A 72-year-old man describes pain about the waist at the level of the umbilicus. The pain is often burning and occasionally shooting. It does not extend down his legs, but he has noticed some weakness in his legs at the time of the pain. With exertion, such as walking, he develops pain in his legs and a tingling sensation in his feet. He has been taking aspirin for the discomfort, but has noticed no substantial change in the sensation. X-rays of his spine revealed no abnormalities. Pain and weakness have become increasingly frequent over the course of several months. Because the man has had urinary hesitancy and frequency in association with an enlarged prostate, he is advised to have a transurethral prostatectomy. A general anesthetic is given for the surgery. On recovering consciousness postoperatively, the man cannot move his legs and has persistent pain at the level of the umbilicus. His plantar responses are bilaterally extensor. Which of the following is the most appropriate emergency evaluation for this patient? a. Voiding cystometrogram b. Electroencephalogram (EEG) c. Somatosensory evoked potentials (SSEPs) d. Aortogram e. Penile-brachial index (PBI)
The answer is d. This patient has symptoms suggestive of ischemic spinal cord disease. The principal source of blood for the spinal cord is the aorta. Vessels that supply the cord are some- what variable in their origins, but they most commonly arise as branches of the vertebral and hypogastric arteries, as well as of the aorta at the level of the upper and lower thoracic vertebrae. The artery most implicated in a patient with this constellation of symptoms is the great anterior medullary artery (of Adamkiewicz), which arises from the aorta at the level of T10-L1 and supplies the anterior median spinal artery.
A 45-year-old left-handed man has had recurrent attacks of "dizzi- ness." He describes the sensation of feeling the room spinning. The episodes occur abruptly and usually last for approximately 45 minutes. The dizziness occurs about once per month, but may happen more frequently. There is often accompanying ringing and decreased hearing in one ear. Which of the following most accurately describes the early hearing loss in this disease? a. Over all frequencies b. Primarily over high frequencies c. Primarily over middle frequencies d. Primarily over low frequencies e. In virtually no patients
The answer is d. Unlike the deficit of presbycusis, lower tones are most susceptible to impaired perception during the initial phases of Ménière's disease. The severity of the hearing loss typically fluctuates considerably. As fluctuations in the low-tone loss abate, high tones become progressively more involved. The attacks of vertigo associated with Ménière's disease usually abate as hearing loss in the affected ear peaks.
A 9-year-old girl presents with precocious puberty and episodes of uncontrollable laughter. Which of the following mass lesions might explain her symptoms? a. Craniopharyngioma b. Choroid plexus papilloma c. Giant aneurysm d. Metastatic carcinoma e. Hypothalamic hamartoma
The answer is e. Hypothalamic hamartomas are non-neoplastic malformations involving neurons and glia in the region of the hypothalamus. They may be discovered incidentally, either on imaging performed for other reasons or at autopsy, or they may cause symptoms referable to the hypothalamus. Most often, the latter involves neuroendocrine functions, causing precocious puberty or acromegaly due to overproduction of growth hormone-releasing hormone. Patients may also experience paroxysms of laughter, known as gelastic seizures. They may be cured surgically. Craniopharyngiomas are epithelial neoplasms arising in the sellar and third ventricular regions. They may cause hypopituitarism and visual field disturbances. Choroid plexus papillomas usually develop intraventricularly and do not extend down into the sella turcica. These tumors affect both children and adults, but they are rare. They are benign if they are surgically accessible and are extirpated early in their evolution. Giant aneurysms occur in many locations, but typically do not cause gelastic seizures or precocious puberty. Metastatic carcinoma generally occurs in older patients and would not be expected to cause these symptoms.
A patient with amyotrophic lateral sclerosis develops progressive difficulty breathing. His cough becomes totally ineffective for clearing his airway, and he requires a tracheostomy. Facial muscle weakness and fascic- ulations are obvious at the time the tracheostomy is performed. Which of the following is the most appropriate treatment for this patient? a. Atropine sulfate b. Pyridostigmine c. Edrophonium d. Amantadine e. Chest physical therapy
The answer is e. This patient has a motor neuron disease. Pyridostigmine and edrophonium are useful in the evaluation and management of neuromuscular junction disease (e.g., myasthenia gravis). Amantadine is useful in the management of Parkinson's disease and MS, improving mobility in the former and reducing fatigue in the latter. Atropine might be of some use in this patient if he has excessive pulmonary secretions, but conscientious pulmonary toilet performed by an experienced physical therapist is much more likely to be beneficial.
An otherwise healthy young woman has poorly responsive pupils that are dilated. Visual acuity is normal. A careful neurologic examination reveals bilaterally absent Achilles tendon jerks. Which of the following is the most likely diagnosis? a. A cervical spinal cord tumor b. A brainstem glioma c. MS d. A posterior communicating artery aneurysm e. Benign tonic pupillary dilatation
The answer is e. This pupillary abnormality is called Adie's tonic pupil. It is usually seen in *otherwise healthy young women and may occur in isolation or in association with absent tendon reflexes*. Local trauma to the eye should be considered if only one pupil is affected. If both pupils are affected, drug use should be considered. It is probably due to degeneration of the ciliary ganglia. Although the cause is obscure, it may reflect a mild polyneuropathy. In most cases this is a benign phenomenon.
A 37-year-old man develops involuntary twitching movements in his left thumb. Within 30 s, he notices that the twitching has spread to his entire left hand and that involuntary movements have developed in his left forearm and the left side of his face. He cannot recall what happened sub- sequently, but his wife reports that he fell down and the entire left side of his body appeared to be twitching. He appeared to be unresponsive for about 3 min and confused for another 15 min. During the episode, he bit his tongue and wet his pants.
The answer is f. (Victor, pp 337-338.) With a Jacksonian march, or sequential seizure, the patient develops focal seizure activity that is primarily motor and spreads. This type of seizure often secondarily generalizes, at which point the patient loses consciousness and may have a generalized tonic-clonic seizure. The hand is a common site for the start of a Jacksonian march. The face may be involved early because the thumb and the mouth are situated near each other on the motor strip of the cerebral cortex.
A 13-year-old girl has headaches and diplopia. On examination, she has impaired upward gaze, lid retraction, and convergence-retraction nys- tagmus. Her pupils react on convergence but not to light.
The answer is h. (Victor, pp 705-708.) Pineocytomas are histologi- cally benign lesions affecting the region of the pineal gland. They arise from the parenchymal cells of the pineal gland. This patient's symptoms and signs constitute Parinaud syndrome, which may include loss of verti- cal gaze, loss of pupillary light reflex, lid retraction, and convergence- retraction nystagmus, in which the eyes appear to jerk back into the orbit on attempted upgaze. This syndrome occurs in lesions due to involvement of the dorsal midbrain in the region of the superior colliculus. Other tumors appearing in the pineal region that can produce a similar clinical picture include germ cell tumors (germinomas), teratomas, and gliomas. Malignant pineal tumors, or pineoblastomas, may also occur, and are sim- ilar histologically to medulloblastomas.
A 50-year-old woman is found wandering in the street and is brought to the emergency room by the police. She is disoriented to time, place, and person, but has no evidence of head trauma. She staggers when she tries to walk, but she has no detectable alcohol in her blood. Eye movements are abnormal with paresis of conjugate gaze, and horizontal nystagmus is apparent. Relatives are contacted, and they report that this woman has a long history of alcohol abuse.
The answer is f. An apparently acute deterioration in cognitive function in an alcoholic may stem from any one of several causes. Bleeding from esophageal varices may have produced a profound anemia. Inapparent head trauma may have produced a subarachnoid hemorrhage or subdural hematomas. If the patient's problem is due to a nutritional deficiency, it is most likely *thiamine (B1) deficiency* associated with alcoholism. That this patient has no alcohol in her blood at the time of the deterioration is irrelevant. The triad of dementia, gait difficulty, and oculomotor paresis is characteristic of Wernicke's encephalopathy, the rapidly progressive and potentially lethal form of thiamine deficiency. Peripheral neuropathy commonly develops with thiamine deficiency, but it is not a component of the encephalopathy caused by thiamine deficiency.
A 17-year-old boy reports involuntary jerking movements in his arms when he awakens. This has occurred during the day after a nap as well as in the morning after a full night's sleep. Over the next few months, he developed similar jerks during the day, even when he had been awake for several hours. He did not lose consciousness with these muscle jerks, but did occasionally fall. On one occasion, jerks in his legs caused a fall that resulted in a fractured wrist.
The answer is j. (Victor, p 109.) Myoclonic seizures may be general- ized or partial. They are most commonly seen in the epilepsy syndrome called benign juvenile myoclonic epilepsy (BJME). Unlike sleep myoclonus, the episodes occur when the affected person wakes up rather than when he or she is falling asleep. Myoclonic jerks may be triggered by light flashes or loud sounds. Benign juvenile myoclonic epilepsy accounts for 4% of all cases of epilepsy. More than half of those with BJME have generalized tonic-clonic seizures as well as myoclonic seizures.
A 7-month-old boy develops generalized limb extension and neck flexion spasms that occur more than 20 times daily and are associated with altered consciousness. EEG reveals diffuse, high-voltage, polyspike-and- slow-wave discharges between spasms and suppression of these bursts during the spasms. A sibling died with a brainstem glioma, and the father has several large areas of hypopigmented skin in the shape of ash leaves. The infant had obvious psychomotor retardation even before the appear- ance of the spasms.
The answer is j. (Victor, p 342.) This child has West syndrome, a generalized seizure disorder of infants characterized by recurrent spasms, the EEG pattern of hypsarrhythmia, and retardation. Several different diseases cause West syndrome. The family history in this case suggests tuberous sclerosis as the underlying problem. Adrenocorticotropic hormone is the best of the given choices.
A 23-year-old woman with a history of hemophilia notices progressive memory difficulty. She has required little hematologic support, but she did receive transfusion of factor VIII at least five times over the past 7 years. Neurologic examination reveals word-finding difficulty, poor recent and remote memory, gait ataxia, mild dysarthria, and a labile affect. Her right plantar response is extensor and her left brachioradialis reflex is hyperactive with transient clonus. An MRI of the brain is unrevealing. Dx?
The answer is n. This woman is at relatively high risk for AIDS encephalopathy because she has required transfusion of clotting factors that have until recently been available only from pooled samples of blood products. The neurologic deficits that she exhibits are not specific for HIV-1-associated subacute encephalomyelitis (AIDS encephalitis) and are quite compatible with multiple sclerosis (MS). That her MRI does not reveal plaques of demyelination scattered throughout the brain makes the diagnosis of MS improbable. To establish the diagnosis of AIDS encephalopathy, HIV-1 antibodies should be sought and the helper/ suppressor (CD4/CD8) T lymphocyte ratio should be checked. Patients with symptomatic AIDS usually have a CD4/CD8 T lymphocyte ratio of less than 0.5.
A 35-year-old businessman has sleep attacks. He runs a chain of dry- cleaning stores, but does not usually work with the cleaning fluids. He reports falling asleep several times during the workday, even at business meetings and during interviews. He has developed the sleep attacks only after gaining over 100 lb. His weight at the time of the examination is 324 lb. Dx?
The answer is o. Obesity associated with hypersomnia qualifies as *pickwickian syndrome* if the patient exhibits other characteristic features, such as sleep apnea. The patient with this syndrome is likely to have hypoxemia and pulmonary hypertension. Smoking increases the risk of developing the syndrome. Sleep attacks usually abate with cessation of smoking and weight loss.
A 50-year-old woman began having double vision and blurry vision 3 months ago and has since had diminishing interaction with her family, a paucity of thought and expression, and unsteadiness of gait. Her whole body appears to jump in the presence of a loud noise. An MRI scan and routine CSF examination are unremarkable. Dx?
The neurological symptoms occurring early in the course of *Creutzfeldt-Jakob* disease are often cerebellar or visual. Patients may have ataxia, clumsiness, or dysarthria, as well as diplopia, distorted vision, blurred vision, field defects, changes in color perception, and visual agnosia. Ultimately, cortical blindness may occur. The diagnosis may be supported by the finding of periodic sharp waves at a 1- to 2-Hz frequency on EEG and the finding of elevated protein 14-3-3 in CSF.The typical EEG pattern is found in up to 80% of patient sat some point during the course of the illness. An MRI may show a pattern of increased T2 signal in the basal ganglia or other gray matter in many, but not all, cases.
A 72-year-old man awakens with severe vertigo associated with nausea and vomiting. He is ataxic. Over the next several days, he develops numbness of the left side of his body, dysphagia, and hiccups. On examination he has a left homonymous hemianopsia, left-sided sensory loss, dysmetria with the right hand, and no weakness. He has had intermittent episodes of dizziness for the past month.
This patient has a history of progressive vertigo, ataxia, sensory loss, dysphagia, and hiccups, all symp- toms of the lateral medullary syndrome, usually due to distal vertebral artery occlusion. This patient's hemianopsia reflects the probable occurrence of occipital lobe infarction, perhaps related to embolism from the occluded vertebral artery. This could have occurred at the time of the lateral medullary stroke or at an independent time. The preceding history of dizzy episodes is indicative of the importance of a thorough evaluation for the cause of dizziness in the elderly patient, particularly when other symptoms occur as well.